Απλή άσκηση juniors

Συντονιστές: Φωτεινή, silouan

Άβαταρ μέλους
∫ot.T.
Δημοσιεύσεις: 49
Εγγραφή: Πέμ Μαρ 23, 2023 4:21 pm
Τοποθεσία: Λουτράκι

Απλή άσκηση juniors

#1

Μη αναγνωσμένη δημοσίευση από ∫ot.T. » Τετ Μάιος 17, 2023 10:44 pm

Αν για τους θετικούς αριθμούς a,b,c ισχύει ότι
\frac{(\frac{a}{b})^{3}+(\frac{b}{c})^{3}+(\frac{c}{a})^{3}}{3}\leq 1
Να βρεθεί το άθροισμα: \frac{a}{b}+\frac{b}{c}+\frac{c}{a}
τελευταία επεξεργασία από ∫ot.T. σε Πέμ Μάιος 18, 2023 5:09 pm, έχει επεξεργασθεί 1 φορά συνολικά.


«Ο μορφωμένος διαφέρει από τον αμόρφωτο, όπως ο ζωντανός από τον νεκρό.» Αριστοτέλης

Λέξεις Κλειδιά:
Άβαταρ μέλους
george visvikis
Επιμελητής
Δημοσιεύσεις: 13301
Εγγραφή: Παρ Νοέμ 01, 2013 9:35 am

Re: Απλή άσκηση juniors

#2

Μη αναγνωσμένη δημοσίευση από george visvikis » Πέμ Μάιος 18, 2023 12:01 am

∫ot.T. έγραψε:
Τετ Μάιος 17, 2023 10:44 pm
Αν για τους άνισους θετικούς αριθμούς a,b,c ισχύει ότι
\frac{(\frac{a}{b})^{3}+(\frac{b}{c})^{3}+(\frac{c}{a})^{3}}{3}\leq 1
Να βρεθεί το άθροισμα: \frac{a}{b}+\frac{b}{c}+\frac{c}{a}
Είναι γνωστό ότι αν x, y,z \ge 0, τότε \displaystyle {x^3} + {y^3} + {z^3} \geqslant 3xyz. Έτσι η δοθείσα σχέση γράφεται:

\displaystyle {\left( {\frac{a}{b}} \right)^3} + {\left( {\frac{b}{c}} \right)^3} + {\left( {\frac{c}{a}} \right)^3} = 3\frac{a}{b} \cdot \frac{b}{c} \cdot \frac{c}{a}

κι επειδή οι a,b,c δεν είναι ίσοι μεταξύ τους, θα είναι \boxed{\frac{a}{b} + \frac{b}{c} + \frac{c}{a} = 0}


Κάτι δεν στέκει με την άσκηση. Δεν το πρόσεξα χτες στη βιασύνη μου. Μόλις με ειδοποίησε ο vgreco ότι αφού οι αριθμοί είναι θετικοί δεν μπορεί να έχουν άθροισμα μηδέν.

Έτσι είναι \displaystyle {\left( {\frac{a}{b}} \right)^3} + {\left( {\frac{b}{c}} \right)^3} + {\left( {\frac{c}{a}} \right)^3} > 3 :?:

Εκτός αν αφαιρεθεί η λέξη "άνισοι" από την εκφώνηση, οπότε θα είναι a=b=c=1.


Henri van Aubel
Δημοσιεύσεις: 876
Εγγραφή: Τρί Σεπ 13, 2022 12:01 pm

Re: Απλή άσκηση juniors

#3

Μη αναγνωσμένη δημοσίευση από Henri van Aubel » Πέμ Μάιος 18, 2023 2:59 pm

Αφού  \displaystyle a,b,c> 0\Longrightarrow \left ( \frac{a}{b} \right )^{3}+\left ( \frac{b}{c} \right )^{3}+\left ( \frac{c}{a} \right )^{3}\geq 3 \left ( \ast \right )

Από υπόθεση είναι  \displaystyle \left ( \frac{a}{b} \right )^{3}+\left ( \frac{b}{c} \right )^{3}+\left ( \frac{c}{a} \right )^{3}\leq 3\left ( \ast \ast \right )

Οπότε έχουμε την ισότητα  \displaystyle \left ( \frac{a}{b} \right )^{3}+\left ( \frac{b}{c} \right )^{3}+\left ( \frac{c}{a} \right )^{3}=3

Συνεπώς έχουμε ισότητα παντού και a=b=c=1

Η άσκηση μπορεί να μετασχηματιστεί σε αυτή εδώ:

Να βρείτε όλες τις διατεταγμένες τριάδες \left ( a,b,c \right ) θετικών πραγματικών αριθμών, που είναι τέτοιοι ώστε  \displaystyle \left ( \frac{a}{b} \right )^{3}+\left ( \frac{b}{c} \right )^{3}+\left ( \frac{c}{a} \right )^{3}\leq 3.


Απάντηση

Επιστροφή σε “Άλγεβρα - Επίπεδο Θαλή/Ευκλείδη (Juniors)”

Μέλη σε σύνδεση

Μέλη σε αυτήν τη Δ. Συζήτηση: Δεν υπάρχουν εγγεγραμμένα μέλη και 2 επισκέπτες